Xem mẫu

  1. Hội thảo Khoa học, Sầm Sơn 28-28/09/2019 MỘT SỐ BẤT ĐẲNG THỨC HÌNH HỌC TRONG TAM GIÁC Nguyễn Văn Ngọc Trường Đại học Thăng Long Tóm tắt nội dung 1 Mở đầu Bất đẳng thức hình học nói chung và vất đẳng thức trong tam giác nói riêng là chuyên mục khó trong lĩnh vực Toán Phổ thông, nhưng lại có sức hấp dẫn kỳ lạ, bởi vì các bất đẳng thức này không chỉ có ý nghĩa về nội dung mà còn khá đẹp về hình thức và đòi hỏi nhiều sáng tạo. Bất đẳng thức trong tam giác là chuyên mục lý thú, hấp dẫn nhiều thế hệ những chuyên gia toán học, những người dạy và người học toán trong các trường cấp Trung học ở khắp nới trên Thế giới. Hầu hết các bất đẳng thức trong tam giác hiện nay có trong các tài liệu chuyên khảo đã được tìm ra từ nhiều thế kỷ trước, nhất là Thế kỷ 20. Các bất đẳng thức này có thể tìm thấy trong nhiều tài liệu, thí dụ như các tài liệu [1] và [5]. Nhiều bất đẳng thức trong tam giác mới được tìm ra trong khoảng vài thập niên gần đây (xem, [3, 4, 7,6] và các tài liệu trong đó), nhưng chưa được hệ thống và giới thiệu ở Việt Nam. Mục đích của bài viết này là trình bày một số các bất đẳng thức kinh điển và bất đẳng thức mới trong tam giác, giúp người dạy và học Toán ở bậc phổ thông có thêm tư liệu phục vụ cho công việc giảng dạy và học tập. 2 Bổ trợ 2.1 Các bất đẳng thức kinh điển của dãy số Chúng ta sẽ cần đến các bất đẳng thức quan trọng sau đây Định lý 2.1. (Bất đẳng thức AM − GM).Với n số thực không âm bất kì a1 , a2 , . . . , an , ta có bất đẳng thức a1 + a2 + . . . + a n √ > n a1 .a2 . . . . .an n Đẳng thức xảy ra khi và chỉ khi a1 = a2 = . . . = an 1
  2. Hội thảo Khoa học, Sầm Sơn 28-28/09/2019 Định lý 2.2 (Bất đẳng thức lũy thừa). Với mọi bộ số không âm a1 , a2 , . . . , an và m = 1, 2, . . . ta đều có a1m + a2m + . . . + am a1 + a2 + · · · + a n m   n > n n Định lý 2.3 (Bất đẳng thức Cauchy - Schawrz). Xét hai bộ số thực tùy ý a1 , a2 , · · · , an và b1 , b2 , · · · , bn . Khi đó ta có ( a1 b1 + a2 b2 + · · · + an bn )2 6 ( a21 + a22 + · · · + a2n )(b12 + b22 + · · · + bn2 ), a1 a2 an Đẳng thức xảy ra khi và chỉ khi = = ··· = . b1 b2 bn (Với quy ước nếu mẫu bằng 0 thì tử cũng bằng 0). Định lý 2.4 (Minkovskii). Với hai bộ số thực { ak }nk=1 , {bk }nk=1 có bất đẳng thức s s n q  n 2  n 2 ∑ a2k + bk2 ≤ ∑ ak + ∑ bk . k =1 k =1 k =1 2.2 Các đại lượng và định lý thông dụng của một tam giác Trong phần này ta luôn giả sử tam giác ABC có: • BC = a, CA = b, AB = c; • S là diện tích tam giác; • p là nửa chu vi tam giác; • m a , mb , mc , wa , wb , wc , h a , hb , hc lần lượt là độ dài các trung tuyến, các phân giác và các đường cao tương ứng với các cạnh a, b, c; • r, R, r a , rb , rc lần lượt là các bán kính đường tròn nội tiếp, đường tròn ngoại tiếp, đường tròn bàng tiếp với các cạnh a, b, c của tam giác ABC. • ∑ a = a + b + c. Định lý 2.5 (Định lý hàm số sin). Trong mọi tam giác ABC có hệ thức a b a = = = 2R. sin A sin B sin A Định lý 2.6 (Định lý hàm số cosin). Trong mọi tam giác ABC có hệ thức a2 = b2 + c2 − 2bc cos A, b2 = c2 + a2 − 2ca cos B, c2 = a2 + b2 − 2ab cos C. Như một hệ quả của Định lý hàm số cosin, ta có khẳng định Địnhnlý Pitago nổi tiếng Định lý 2.7 (Định lý Pitago). Tam giác ABC vuông tại A khi và chỉ khi a2 = b2 + c2 . 2
  3. Hội thảo Khoa học, Sầm Sơn 28-28/09/2019 Định lý 2.8 (Định lý Apollonius-Pappus). Tam giác ABC có các hệ thức sau đây về đường trung tuyến 2b2 + 2c2 − a2 m2a = , 4 2c2 + 2a2 − b2 m2b = , 4 2a2 + 2b2 − c2 m2c = . 4 Định lý 2.9 (Định lý đường phân giác). Tam giác ABC có các hệ thức sau đây về các đường phân giác 2bc 2ca 2ab wa = , wb = , wc = . b+c c+a a+b Định lý 2.10 (Về diện tích của tam giác). Diện tích S của tam giác ABC được tính theo các công thức 1 1 1 S= ah a = bhb = chc 2 2 2 1 1 1 = ab sin C = bc sin A = ca sin B 2 2 2 abc = 4R = sr = ( p − a )r a = ( p − b )r b = ( p − c )r c q = p( p − a)( p − b)( p − c), ( Công thức Heron ). Định lý 2.11 (Bán kính đường tròn nội tiếp). Trong tam giác ABC ta có A B C r = ( p − a) tan = ( p − b) tan = ( p − c) tan . 2 2 2 Định lý 2.12 (Bán kính đường tròn bàng tiếp). Trong tam giác ABC ta có A B C r a = p tan , rb = p tan , rc = p tan . 2 2 2 Định lý 2.13 (Công thức Euler). Trong tam giác ABC ta có R( R − 2r ) = OI 2 , trong đó O và I tương ứng ký là tâm của đường tròn ngoại tiếp và tâm của đường tròn nội tiếp tam giác ABC. 3
  4. Hội thảo Khoa học, Sầm Sơn 28-28/09/2019 3 Một số bất đẳng thức kinh điển liên quan đến đường cao và đường trung tuyến Mệnh đề 3.1. Trong mọi tam giác có bất dẳng thức 9 9r ≤ h a + hb + hc ≤ R. (3.1) 2 Chứng minh. Từ công thức của diện tích ram giác ta có 2sr 2sr 2sr ha = , hb = , hc = . a b c Suy ra 1 1 1 h a + hb + hc = r ( a + b + c) + + ≥ 9r. a b c Do đó vế trái của (3.1) được chứng minh. Cũng từ công thức diện tích của tam giác ta có bc ac ab ha = , hb = , , hc = . 2R 2R 2R Suy ra bc + ca + ab a2 + b2 + c2 h a + hb + hc = ≤ . 2R 2R Theo Định lý hàm số sin ta có a2 + b2 + c2 9 = 2R(sin2 A + sin2 B + sin2 C ) ≤ R. 2R 2 Vậy vế phải của (3.1) cũng được chứng minh. Mệnh đề được chứng minh. Mệnh đề 3.2. Trong mọi tam giác ABC có bất dẳng thức √ 3 h a + hb + hc ≤ ( a + b + c ). (3.2) 2 Dấu đẳng thức xảy ra khi và chỉ khi tam giác ABC là đều. Chứng minh. Ta có bất đẳng thúc 3abc ( a + b + c)2 ≥ 3(bc + ca + ab) = (h a + hb + hc ) = 6R(h a + hb + hc ). 2S Mặt khác lại có ( a + b + c )2  1 ≤ ( a2 + b2 + c2 ) = 8R2 (1 + cos A cos B cos C ) = 8R2 1 + = 9R2 . 3 8 Suy ra √ 6 a+b+c ≤ 3R, ( a + b + c)2 ≥ √ ( a + b + c)(h a + hb + hc ). 3 3 Từ đây ta có điều phải chứng minh. 4
  5. Hội thảo Khoa học, Sầm Sơn 28-28/09/2019 Mệnh đề 3.3. Trong mọi tam giác ABC có bất đẳng thức m a + mb + mc ≤ 4R + r. (3.3) Dấu đẳng thức xảy ra khi và chỉ khi tam giác ABC là đều. Chứng minh. Ký hiệu p a , pb , pc tương ứng là khoảng cách từ tâm vòng tròn ngoại tiếp tam giác ABC đến các cạnh BC, CA, AB. Khi đó m a ≤ R + p a , mb ≤ R + pb , mc ≤ R + pc . Theo công thức tính diện tích của tam giác, ta có ap a = R2 sin 2A, bpb = R2 sin 2B, cpc = R2 sin 2C. Suy ra  r p a + pb + pc = R(cos A + cos B + cos C ) = R 1 + . R Vậy ta có r  m a + mb + mc ≤ 3R + R 1 + = 4R + r. R Mệnh đề được chứng minh. Mệnh đề 3.4. Trong mọi tam giác ABC có bất đẳng thức √ m2a + m2b + m2c ≥ 3 3S. (3.4) Dấu đẳng thức xảy ra khi và chỉ khi tam giác ABC là đều. Chứng minh. Ta có 3 2 m2a + m2b + m2c = ( a + b2 + c2 ). 4 Chúng ta chứng minh bất đẳng thức Weisenbock √ a2 + b2 + c2 ≥ 4 3S. Thật vậy, ta có √ √ 1 a2 + b2 + c2 − 4 3S = 2(b2 + c2 ) − 2bc cos A − 4 3 sin A 2 1 √ 2 2 3  π = 2(b + c ) − 4bc cos A + sin A = 2(b2 + c2 ) − 4bc cos( A − ) 2 2 3 ≥ 2(b2 + c2 ) − 4bc = 2(b − c)2 ≥ 0. π Dấu đẳng thức xảy ra khi và chỉ khi b = c và A = , tức tam giác ABC đều. Vậy ta có 3 3 2 3 √ √ m2a + m2b + m2c = ( a + b2 + c2 ) ≥ 4. 3S = 3 3S. 4 4 Mệnh đề được chứng minh. 5
  6. Hội thảo Khoa học, Sầm Sơn 28-28/09/2019 4 Bất đẳng thức Brown-Barrero về đường trung tuyến Mục này trình bày một bất đẳng thức về đường trung tuyến do Brown p.H. và Diaz- Barrero J.L. nhận được trong [2]. Trước hết ta cần Bổ đề sau đây. Bổ đề 4.1. Cho ABC là tam giác có độ dài các cạnh là a, b, c và bán kính đường tròn nội tiếp là r. Khi đó a2 ( b + c ) abc ∑ (a + b)(a + c) ≤ 8r2 . (4.1) quay vòng a+b+c Chứng minh. Để chứng minh bất đẳng thức trên, chúng ta thêm số hạng p = 2 và sẽ chỉ ra a2 ( b + c ) a+b+c abc ∑ ( a + b)( a + c) ≤ 2 ≤ 2. 8r (4.2) quay vòng abc Từ công thức đã biết S = = pr kết hợp với bất đẳng thức Euler R ≥ 2r, ta có 4R abc abc p= ≤ 2 4Rr 8r và vế phải của bất đẳng thức được chứng minh. Để chứng minh vế trái của bất đẳng thức, ta chú ý rằng 2a2 (b + c) ∑ (a + b)(a + c) ≤ a + b + c. (4.3) quay vòng Thật vậy, ta có a2 ( b + c ) 4( a2 b2 + b2 c2 + c2 a2 + a2 bc + ab2 c + abc2 ) ∑ ( a + b)( a + c) = ( a + b)(b + c)(c + a) quay vòng ( a + b + c)( a + b)(b + c)(c + a) − ab( a − b)2 − bc(b − c)2 − ca(c − a)2 = ( a + b)(b + c)(c + a) ab( a − b) + bc(b − c)2 + ca(c − a)2 2 = ( a + b + c) − ≤ a+b+c ( a + b)(b + c)(c + a) như mong muốn. Dấu bằng xảy ra nếu a = b = c. Tức là, khi tam giác là tam giác đều. Bổ đề được chứng minh. Định lý 4.1. Cho ABC là tam giác có độ dài các trung tuyến ứng với các cạnh là m a , mb b, mc và diện tích S. Khi đó m2a (mb + mc ) m5 + m5b + m5c ∑ (m a + mb )(m a + mc ) ≤ a 6S2 . (4.4) quay vòng Chứng minh. Trong bất đẳng thức (4.1) ta đặt 2 2 2 a= m a , b = mb , c = mc , 3 3 3 6
  7. Hội thảo Khoa học, Sầm Sơn 28-28/09/2019 sử dụng công thức S r= m a + mb + mc và bất đẳng thức trung bình AM-GM, ta có m2a (mb + mc ) ( m a + m b + m c )2 ( m a m b m c ) ∑ (m a + mb )(m a + mc ) ≤ 18S2 quay vòng m a + mb + mc 2 m a + mb + mc 3 m5 + m5b + m5c     1 ≤ 2 ≤ a . 2S 3 3 6S2 Dấu bằng xảy ra khi m a = mb = mc . Tức là, khi 4 ABC là tam giác đều, suy ra điều phải chứng minh. 5 Bất đẳng H.Y.Yin mở rộng 5.1 Giới thiệu Mục này trình bày các kết quả của Jian Liu [4] chứng minh một bất đẳng thức đã biết liên quan đến cận trên của tổng đường trung tuyến là mạnh. Tác giả cũng đã chứng minh một giả thiết mạnh hơn về bất đẳng thức này, tương đương với bất đẳng thức được đề xuất bởi H.Y.Yin trong [1]. Cho ABC là tam giác có các đường trung tuyến m a , mb , mc và nửa chu vi p, khi đó ta có bất đẳng thức sau (xem [1]): m a + mb + mc < 2p, (5.1) trong đó hằng số 2 là tối ưu. Năm 2000, Chu Xiao-Guang và Yang Xue-Zhi (xem tài liệu trong [4]) thiết lập một bất đẳng thức mới mạnh hơn, được cho bởi Định lý sau đây. Định lý 5.1. Trong tam giác ABC bất kỳ với các đường trung tuyến m a , mb , mc , nửa chu vi p, bán kính đường tròn nội tiếp r và bán kính đường tròn ngoại tiếp R, bất đẳng thức sau đúng: (m a + mb + mc )2 ≤ 4p2 − 16Rr + 5r2 . (5.2) Dấu bằng xảy ra khi và chỉ khi tam giác ABC là tam giác đều. Bất đẳng thức (5.2) là một cận trên xuất sắc cho biểu diễn (m a + mb + mc )2 . Nhưng các tác giả trên vẫn chưa trả lời một câu hỏi tự nhiên là: tìm giá trị cực đại λ sao cho bất đẳng thức (m a + mb + mc )2 ≤ 4p2 − λRr + (2λ − 27)r2 . (5.3) đúng với mọi tam giác ABC. Một trong các mục tiêu của bài báo này là chứng minh kết luận liên quan sau: Định lý 5.2. Giả sử λ là số thực dương sao cho bất đẳng thức (5.3) đúng với mọi tam giác ABC. Khi đó λmax = 16. Bất đẳng thức (5.3) trở thành (5.2) khi k = 16. Điều này có nghĩa bất đẳng thức (5.2) là mạnh nhất trong tất cả các bất đẳng thức dạng (5.3). 7
  8. Hội thảo Khoa học, Sầm Sơn 28-28/09/2019 5.2 Chứng minh của Định lý 5.2 Chứng minh. Nếu R 6= 2r, thì bất đẳng thức (5.3) tương đương với 4p2 − 27r2 − (m a + mb + mc )2 λ≤ . r ( R − 2r ) Theo các đẳng thức đã biết pr2 = ( p − a)( p − b)( p − c), abc = 4Rr, ta thấy thêm rằng bất đẳng thức trên tương đương với 4p3 − 27( p − a)( p − b)( p − c) − p(ma + mb + mc)2 λ≤ . (5.4) abc − 8( p − a)( p − b)( p − c) Bây giờ ta giả sử ABC là tam giác cân với các cạnh x, 1, 1 ( x 6= 1), đặt b = c = 1, a = x, 1√ 2 rồi dùng công thức trung tuyến m a = 2b + 2c2 − a2 và các bất đẳng thức đã biết: 2 s 1 abc ( p − a)( p − b)( p − c) p = ( a + b + c ), R = p ,r = , 2 4 ( p − a)( p − b)( p − c) p ta được 1 1p 1p p= x + 1, m a = 4 − x2 , mb = mc = 1 + 2x2 2 2√ 2 1 x 4 − x2 R= √ ,r = . 4 − x2 2( x + 2) Thay a = x, b = c = 1 và các hệ thức trên vào (5.4), sau một vài tính toán ta được p 12x3 − 22x2 + 20x − 2( x + 2) (4 − x2 )(1 + 2x2 ) + 8 λ≤ . (5.5) x ( x − 1)2 Đặt q 3 2 f ( x ) = 12x − 22x + 20x − 2( x + 2) (4 − x2 )(1 + 2x2 ) + 8, g( x ) = x ( x − 1)2 . Thì suy ra p 0 12x4 + 16x3 − 28x2 − 28x − 8 + 4(9x2 − 11x + 5) (4 − x2 )(1 + 2x2 ) f (x) = p (4 − x2 )(1 + 2x2 ) g0 ( x ) = 3x2 − 4x + 1. Lưu ý rằng lim f ( x ) = 0, lim g( x ) = 0. x →0 x →0 Theo bất đẳng thức (5.5) và quy tắc L’Hopital, ta được f (x) f 0 (x) λ ≤ lim = lim 0 x →0 g( x ) x →0 g ( x ) p 12x4 + 16x3 − 28x2 − 28x − 8 + 4(9x2 − 11x + 5) (4 − x2 )(1 + 2x2 ) = lim p x →0 (3x2 − 4x + 1) (4 − x2 )(1 + 2x2 ) = 16. Do đó ta chứng minh xong định lý 5.2. 8
  9. Hội thảo Khoa học, Sầm Sơn 28-28/09/2019 Nhận xét 5.1. Chu Xiao-Guang và Yang Xue-Zhi (xem tài liệu trong [4]) cũng thu được bất đẳng thức ngược của (5.2): (m a + mb + mc )2 ≥ 4p − 28Rr + 29r2 . (5.6) Theo cách tương tự như chứng minh của Định lý 5.2, ta cũng chứng minh được kết luận sau: Cho λ là số thực dương thỏa mãn (m a + mb + mc )2 ≥ 4p − λRr + (2λ − 27)r2 . (5.7) với mọi tam giác ABC, khi đó λmin = 28. 6 Bất đẳng thức mới về tổng các đường trung tuyến Trong mục này, chúng tôi giới thiệu một bất đẳng thức mới về tổng của các đường trung tuyến trong tam giác và hai bất đẳng bất đẳng thức khác được chứng minh bằng cách sử dụng bất đẳng thức này [3]. Định lý 6.1. Trong mọi tam giác ABC với các cạnh a, b, c, đường trung tuyến m a , mb , mc , bán kính đường tròn nội tiếp r, bán kính đường tròn ngoại tiếp R, bất đẳng thức sau đúng: ( m a + m b + m c )2 r2 2 2 2 ≤ 2 + 2, (6.1) a +b +c R với dấu bằng xảy ra khi và chỉ khi 4 ABC là tam giác đều. Gần đây, bất đẳng thức (6.1) đã được chứng minh bởi Liu J.(xem, [3, 4]). Tuy nhiên, chứng minh này rất phức tạp. Tác giả dùng một số bổ đề , trong đó có bất đẳng thức 4p( p − a)(b − c)2 4mb mc ≤ 2a2 + bc − , (6.2) 2a2 + bc với dấu bằng xảy ra khi và chỉ khi b = c. Chứng minh của Định lý 6.1 khá dài, nên chúng tôi không trình bày ở đây. Trong mục này, ta sẽ áp dụng Định lý 6.1 để thiết lập hai bất đẳng thức mới, mà vẫn chưa được chứng minh bằng cách dùng bất đẳng thức (5.2). Định lý 6.2. Với mọi tam giác ABC ta có: m a + mb + mc − (h a + hb + hc ) ≤ 2( R − 2r ), (6.3) với dấu bằng xảy ra khi và chỉ khi 4 ABC là tam giác đều. Chứng minh. Theo Định lý 6.1, để chứng minh (6.3) ta cần chứng minh r2   ( a + b + c ) 2 + 2 ≤ [h a + hb + hc + 2( R − 2r )]2 . 2 2 2 (6.4) R Nhân cả hai vế với 4R2 và dùng hệ thức 2Rh1 = bc, v.v., bất đẳng thức (6.4) trở thành dạng tương đương sau: M0 ≡ [bc + ca + ab + 4R( R − 2r )]2 − 4( a2 + b2 + c2 )(2R2 + r2 ) ≥ 0. (6.5) 9
  10. Hội thảo Khoa học, Sầm Sơn 28-28/09/2019 Áp dụng đẳng thức [5] ∑ a2 = 2p2 − 8Rs − 2r2 và đẳng thức đã biết bc + ca + ab = p2 + 4Rr + r2 , (6.6) dễ dàng thu được M0 = (4R2 + 4Rr + 3r2 − p2 )2 . Từ đó rút ra bất đẳng thức M0 ≥ 0 và (6.3) được chứng minh. Rõ ràng dấu bằng trong (6.3) chỉ xảy ra khi 4 ABC là tam giác đều. Định lý được chứng minh. Nhận xét 6.1. Bằng phương pháp chứng minh Định lý 2 trong [4], ta có thể chứng minh rằng hằng số 2 ở vế phải của (6.3) là tối ưu. Thêm vào đó, từ bất đẳng thức Leuenberger [1]: h a + hb + hc ≤ 2R + 5r, (6.7) ta thấy rằng bất đẳng thức (6.3) mạnh hơn kết quả sau: m a + mb + mc ≤ 4R + r. (6.8) Nhận xét 6.2. Bằng cách sử dụng bất đẳng thức (5.2), ta dễ dàng chứng minh bất đẳng thức tuyến tính cho tổng m a + mb + mc : √ m a + mb + mc ≤ 2p − (6 3 − 9)r. (6.9) Bất đẳng thức này cũng mạnh hơn (6.8) vì ta có bất đẳng thức sau: √ p ≤ 2R + (3 3 − 4)r (6.10) của W.J. Blundon (xem tài liệu trong [3]). Tiếp theo, ta chứng minh một bất đẳng thức về tam giác nhọn, mà được tác giả tìm ra trong nhiều năm trước nhưng vẫn chưa chứng minh được cho tới bây giờ. Định lý 6.3. Cho tam giác nhọn ABC ta có: h a + hb + hc 1 r ≥ + . (6.11) m a + mb + mc 2 R Dấu bằng xảy ra khi và chỉ khi tam giác nhọn ABC là tam giác đều. Chứng minh. Theo Định lý 6.1, để chứng minh (6.11), ta chỉ cần chứng minh rằng r 2 r2     2 1 ( h a + hb + hc ) − + 2 + 2 ( a2 + b2 + c2 ) ≥ 0. (6.12) 2 R R Nhân cả hai vế bất đảng thức trên rồi dùng hệ thức 2Rh a = bc, v.v., ta thấy (6.12) tương đương với N0 ≡ R2 (bc + ca + ab)2 − ( R + 2r )2 (2R2 + r2 )( a2 + b2 + c2 ) ≥ 0. (6.13) Thay (??) và (6.6) vào biểu thức của N0 , thì (6.13) tương đương với N0 ≡ p4 R2 − 4( R4 + 2R3 r + 4R2 r2 + 2Rr3 + 2r4 ) p2 (6.14) + (4R + r )(4R4 + 20R3 r + 19R2 r2 + 8Rr3 + 8r4 ) ≥ 0. 10
  11. Hội thảo Khoa học, Sầm Sơn 28-28/09/2019 Ta viết lại N0 như sau N0 = 4r2 ( R + 2r )(2R2 + r2 )e + 8r4 G2 + R[4r (3R + r )eRG1 ]C0 , (6.15) trong đó e = R − 2r G1 = p2 − 16Rr + 5r2 G2 = 4R2 + 4Rr + 3r2 − p2 C0 p2 − (2R + r )2 . Do đó theo bất đẳng thức Euler e ≥ 0, bất đẳng thức Gerretsen G1 ≥ 0, G2 ≥ 0, và bất đẳng thức tam giác nhọn C0 ≥ 0 của Ciamberlini, ta kết luận rằng N0 ≥ 0 đúng cho tam giác nhọn ABC. Nên bất đẳng thức (6.12) và (6.11) được chứng minh. Dễ kiểm tra được dấu bằng trong (6.11) xảy ra khi 4 ABC là tam giác đều. Đinhj lý được chứng minh. 7 Bất đẳng thức mới về tổ hợp tuyến tính của các đường cao và đường trung tuyến Mục này trình bày một số bất đẳng thức về tổ hợp tuyến tính của các đường cao và đường trung tuyến dựa trên các kết quả Zhivko Zhelev trong [6] năm 2008. Hình học về tam giác là một lĩnh vực của hình học cở sở nơi các kết quả mới thú vị luôn bật lên. Có rất nhiều định lý liên quan đến hình học về tam giác, bao gồm hàng trăm các bất đẳng thức hình học (ví dụ xem [1]). Kết quả sau đây bao gồm hai bất đẳng thức mà trông khá dễ chịu, nhưng bất đẳng thức thứ hai hóa ra là vô cùng khó khăn để giải quyết. Thực ra, chúng tôi chứng minh kết quả sau. Định lý 7.1. Cho 4 ABC là tam giác bất kỳ với các cạnh a, b và c. Đặt h a , hb , hc và m a , mb , mc tương ứng là đường cao và trung tuyến của các cạnh. Khi đó hai bất đẳng thức sau đúng: √ √ √ ah a + bhb + chc ≤bch a + achb + abhc (7.1) √ √ √ am a + bmb + cmc ≤ bcm a + acmb + abmc . (7.2) Theo như những gì tác giả biết, bất đẳng thức thứ hai vẫn chưa được chứng minh. Nhưng kiểm tra ngẫu nhiên các giá trị cạnh tam giác không tìm ra phản ví dụ. Chứng minh của chúng tôi dựa trên lý thuyết hàm một biến và hai biến mà chúng tôi cho rằng đây là một trở ngại vì bài toán bên trên thuộc lĩnh vực toán học sơ cấp. Chứng minh của Định lý 7.1. • Để chứng minh bất đẳng thức (7.1) ta sử dụng ah a = bhb = chc = 2S4 ABC . Khi đó √ √ √ ah a + bhb + chc − bch a − achb − abhc √ √ √ 2S bc 2S ac 2S ab = 6S − − − a b c √ √ √ ! bc ac ab = 2S 3 − − − . a b c 11
  12. Hội thảo Khoa học, Sầm Sơn 28-28/09/2019 Do đó, √ √ √ ah a + bhb + chc ≤ bch a + achb + abhc √ √ √ ! bc ac ab ⇔ 3− − − ≤0 a b c 3 3 3 ⇔ 3abc ≤ (bc) 2 + (ca) 2 + ( ab) 2 , a > 0, b > 0, c > 0. √ √ √ Thay x = bc, y = ac, z = ab, ta được 3 3 3 3abc ≤ (bc) 2 + (ca) 2 + ( ab) 2 ⇔ 3xyz ≤ x3 + y3 + z3 , x > 0, y > 0, z > 0. Bất đẳng cuối cùng hiển nhiên đúng theo bất đẳng thức Am-GM và ta chứng minh xong (7.1). • Bây giờ chúng ta sẽ chúng minh bất đẳng thức (7.2). Không giảm tính tổng quát, ta có thể giả sử với 4 ABC rằng a ≥ b ≥ c. Có ba trường hợp có thể: 1. 4 ABC là tam giác đều, tức là a = b = c. 2. 4 ABC là tam giác cân, tức là a = b > c. 3. 4 ABC là tam giác tùy ý, tức là a > b > c. Trước tiên, ta viết lại (7.2) dưới dạng √ √ √ ( a − bc)m a + (b − ac)mb + (c − ab)mc ≤ 0. (7.3) √ √ √ Trường hợp thứ nhất. Nếu 4 ABC là tam giác đều, thì a − bc = b − ac = c − ab = 0 và (7.3) là đẳng thức. Ở bên dưới ta sẽ thấy trong trường hợp này thật ra là trường hợp cực trị của bài toán. Trường hợp thứ hai. Bây giờ cho 4 ABC là tam giác đều ta có thể giả sử a = b > c > 0. Mặt khác ta có 1p 2 1p 2 ma = 2b + 2c2 − a2 = a + 2c2 = mb , 2 2 (7.4) 1p 2 1p 2 mc = 2a + 2b2 − c2 = 4a − c2 . 2 2 Sử dụng (7.4), (7.3) trở thành √ p a − cp 2 (a − ac) a2 + 2c2 ≤ 4a − c2 , a > c. (7.5) 2 12
  13. Hội thảo Khoa học, Sầm Sơn 28-28/09/2019 Ta sẽ chứng minh (7.5). Sau một số tính toán, ta thu được: √ p a − cp 2 ( a − ac) a2 + 2c2 ≤ 4a − c2 , a > c 2 √ √ √ √ √ √ √ p ( a − c)( a + c) p 2 ⇔ a( a − c) a + 2c ≤ 2 2 4a − c2 2 p √ √ p ⇔ 2 a3 + 2ac2 ≤ ( a + c) 4a2 − c2 4( a3 + 2ac2 ) √ 2 2 ≤ a + c + 2 ac 4a − c 4a3 + 8ac2 − 4a3 + ac2 − 4a2 c + c3 √ ≤ 2 ac 4a2 − c2 2 c3  2 2 ⇔ 4a − c ≤ 4ac 9ac2 − 4a2 c c6 + 81a2 c4 + 16a4 c2 + 18ac5 − 8a2 c4 − 72a3 c3 ≤ 64a5 c + 4ac5 − 32a3 c3 ⇔ c6 + 14ac5 + 73a2 c4 − 40a3 c3 + 16a4 c2 − 64a5 c ≤ 0 ⇔ c(c5 + 14ac4 + 73a2 c3 − 40a3 c2 + 16a4 c − 64a5 ) ≤ 0 (c5 + 14ac4 + 73a2 c3 − 40a3 c2 + 16a4 c − 64a5 ) ≤ 0  c 5  c 4  c 3  c 2 c + 14 + 73 − 40 + 16 − 64 ≤ 0. a a a a a c Trong biểu diễn cuối cùng, đặt = t ∈ (0, 1), suy ra rằng a t5 + 14t4 + 73t3 − 40t2 + 16t − 64 ≤ 0 ⇔ (t − 1) t|4 + 15t3 + 88t 2 {z + 48t + 64} ≤ 0. | {z } 0 nếu t>0 Nhưng bất đẳng thức cuối cùng hiển nhiên đúng và chứng minh (7.5) trong trường hợp này.  Trường hợp thứ ba. Bây giờ cho 4 ABC là tam giác tùy ý và cho a > b > c > 0. Ta viết lại (7.3) thành 1 √ p 1 √ p ( a − bc) 2b2 + 2c2 − a2 + (b − ca) 2a2 + 2c2 − b2 2 2 (7.6) 1 √ p 2 2 + (c − ab) 2a + 2b − c ≤ 0 2 2 hay  r r ! s  2   r s  2 1 2 b c b  c  2 b c  c  2 b  a 1− 2 +2 −1+ − 2+2 − 2 a a a a a a a a | {z } F ( ba , ac ):= F ( x,y) r !s  2   1 2 c b b c 2 + a − 2+2 − ≤ 0, 2 a a a a | {z } F ( ba , ac ):= F ( x,y) (7.7) 13
  14. Hội thảo Khoa học, Sầm Sơn 28-28/09/2019 1 và do vậy a2 F ( x, y) ≤ 0 ⇔ F ( x, y) ≤ 0. 2 Chúng tôi gọi hàm hai biến F ( x, y) định nghĩa như bên trên là hàm cá quỷ và bề mặt đồ thị của hàm này trong R3 được gọi là mặt cá quỷ. Theo ký hiệu mới bất đẳng thức (7.3) chuyển thành bài toán cực trị max F ( x, t), trong đó ( x,y)∈ M⊂R2 (7.8) M = {( x, y) ∈ R2 | 0 ≤ y ≤ x ≤ 1, x + y ≥ 1}, trong đó M là tam giác một tam giác vuông và bất đẳng thức cuối cùng được suy ra từ bất đẳng thức a + b > c trong tam giác bất kỳ. Việc kiểm tra tính đúng đắn của hàm cá quỷ trên M là dễ dàng. Do đó, để chứng minh (7.3), ta phải chứng minh max(x,y)∈ M F ( x, y) = 0. Lưu ý rằng M là tập compact và vì M là hàm liên tục trên M, từ các kết quả thông thường trong giải tích thực suy ra rằng F ( x, y) đạt được cực đại ở đây. Để tìm điểm cực đại này ta xét các điểm trong và điểm biên của M, M = intM ∪ ∂M. Dùng các phần mềm toán học, ví dụ như Maple ta thấy rằng p √ ∂F ( x, y) y 2x2 + 2y2 − 1 2(1 − xy) x q =− √ +p + 2 + 2y2 − x2 ∂x 2 xy 2x2 + 2y2 − 1 √ √ ( x − y) x 2 + 2x2 − y2 2( y − x ) x − − √ +p , 2 + 2y2 − x2 2 x 2 + 2x2 − y2 và vì hàm cá quỷ là hàm đối xứng, tức là F ( x, y) = F (y, x ), ta suy ra p √ ∂F ( x, y) x 2y2 + 2x2 − 1 2(1 − xy)y q =− √ +p + 2 + 2x2 − y2 ∂y 2 xy 2y2 + 2x2 − 1 √ √ (y − x )y 2 + 2y2 − x2 2( x − y ) y − − √ + . 2 + 2x2 − y2 p 2 y 2 + 2y2 − x2 Bây giờ ta giải hệ  ∂F   =0 ∂x ∂F   =0 ∂y ta được điểm tới hạn M1 (0, 9238127491 . . . , 0, 1660179102 . . .) và M2 (1, 1). Chú ý M1 , M2 ∈ M và M1 ∈ intM, M2 ∈ ∂M. Một lần nữa dùng phần mềm, ta được ma trận Hessian của hàm cá quỷ:  2 2 ∂2 F ∂2 F ∂ F H ( x, y) := · − ∂x2 ∂y2 ∂x∂y | M2 (1,1) √ ! √ ! √ !4  2 3 3 3 3 3 3 9 = − − − = ≥0 2 2 3 4 √ ∂2 F 3 3 và vì 2 =− < 0, suy ra điểm M2 (1, 1) là điểm cực đại trên M. Để cho đầy ∂x | M2 (1,1) 2 đủ, ta cần kiểm tra điều gì xảy ra trên biên bởi vì M2 ∈ ∂M. Nhưng điều này là dễ dàng 14
  15. Hội thảo Khoa học, Sầm Sơn 28-28/09/2019 và việc kiểm tra này bao gồm các trường hợp { x = 0}, { x = y}, { x = 1} và { x + y = 1}. Điều này dẫn tới kết quả rằng điểm trong câu hỏi thật ra là điểm cực đại tuyệt đối trong M. Điều này kéo theo sup F ( x, y) = max F ( x, y) = F (1, 1) = 0. ( x,y)∈ M ( x,y)∈ M Thêm vào đó, M1 là điểm cực tiểu địa phương và ngoài ra F ( x, y)| M1 = F (0, 9238127491 . . . , 0, 1660179102 . . .) = −0, 4280657968 . . . Mặt cá quỷ có thể nhìn thấy như sau: Sử dụng phần mềm Maple hoặc Mathematica, ta có thể xem được các góc nhìn khác nhau của mặt này: Do đó tất cả các trường hợp của định lý được chứng minh.  Hệ quả sau là đúng. Hệ quả 7.1. Cho a, b và c là các cạnh của tam giác 4 ABC và cho m a , mb , mc tương ứng là đường trung tuyến của các cạnh. Khi đó a) (2p − 3a √)m a +√(2p −√ 3b)mb + (2p − 3c)mc ≥ 0. ma ab + ac + bc b) ≤ ≤ 1, a ≥ b ≥ c. mc a+b+c 8 Một số bất đẳng thức kinh điển liên quan đến đường phân giác Chương này trình bày một số bất đẳng thức hình học kinh điển và các bất đẳng thức mới liên quan đến đường phân giác và đường trung tuyến của tam giác. Nội dung cửa chương này được hình thành chủ yếu từ các tài liêu [1, 7]. Mệnh đề 8.1. Trong tam giác ABC, nếu A wc . Chứng minh. Ta có 4bcp( p − a) 4abp( p − c) w2a = 2 , w2c = . (b + c) ( a + b )2 Suy ra ( a + b )2 ( c + b )2 2 (wa − w2c ) = c(b + c − a)( a + b)2 − a( a + b − c)(b + c)2 2bp = b3 (c − a) + b2 (c2 − a2 ) + 3abc(c − a) + ca(c2 − a2 ). Vì A wc . Mệnh đề được chứng minh. Mệnh đề 8.2. Trong mọi tam giác ABC có bất đẳng thức w a wb + wb wc + wc w a ≤ r a rb + rb r c + r c r a . (8.1) Dấu đẳng thức xảy ra khi và chi khi tam giác ABC đều. 15
  16. Hội thảo Khoa học, Sầm Sơn 28-28/09/2019 Chứng minh. Ta có q q q wa ≤ p ( p − a ), w b ≤ p ( p − b ), w c ≤ p ( p − c ). sc q wa wb ≤ p ( p − a)( p − b) ≤ , ... 2 Suy ra w a w b + w b w c + w c w a ≤ p2 . Mặt khác, từ các công thức S S S ra = , r = , rc = p−a b p−b p−c suy ra r a r b + r b r c + r c r a = p2 . Từ đó ta có điều phải chứng minh. Mệnh đề 8.3. Trong mọi tam giác ABC có bất đẳng thức r a w a + r b w b + r c w c ≤ p2 . (8.2) Dấu đẳng thức xảy ra khi và chỉ khi tam giác ABC là đều. Chứng minh. Từ các công thức s √ q p( p − b)( p − c) 2 bc ra = , wa = p( p − a) p−a b+c suy ra sa q ra wa ≤ p ( p − b)( p − c) ≤ . 2 Vậy ta có a+b+c r a w a + rb wb + rc wc ≤ p = p2 . 2 Mệnh đề được chứng minh. Mệnh đề 8.4. Trong mọi tam giác ABC có bất đẳng thức  r λ  r λ  r λ a c + b + ≥ 3, λ > 0. (8.3) wa wb wc Dấu đẳng thức xảy ra khi và chỉ khi tam giác ABC đều. Chứng minh. Ta có p  r λ a ( p − b)λ ( p − c)λ ≥ , ... wa ( p − a)λ Từ đó suy ra  r λ  r λ  r λ a b c + + wa wb wc p p p ( p − b)λ ( p − c)λ ( p − a)λ ( p − c)λ ( p − b)λ ( p − a)λ ≥ + + ≥3 ( p − a)λ ( p − b)λ ( p − c)λ trên cơ sở của bất đẳng thức AM=GM. Rõ ràng là dấu đẳng thức xảy ra khi và chi khi a=b=c. 16
  17. Hội thảo Khoa học, Sầm Sơn 28-28/09/2019 Mệnh đề 8.5. Trong mọi tam giác ABC có bất đẳng thức √ w2a + w2b + w2c ≥ 3 3S.. (8.4) Dấu đẳng thức xảy ra khi và chỉ khi tam giác ABC đều. Chứng minh. Vì a2 bc bc 1 w2a = bc − 2 , 2 ≤ , (b + c) (b + c) 4 nên 1 ∑ w2a ≥ ∑ ab − 4 ∑ a2 Theo các bất đẳng thức Hadwiger-Finsler và Gordon [1]( sẽ được chúng minh dưới dạng các bồ đề dưới đây) √ √ ∑ ab ≥ 4 3S, ∑ a2 ≤ 2 ∑ ab − 4 3S, ta có 1 √ √ √ √ ∑ w2a ≥ 2 ∑ ab + 3S ≥ 2 3S + 3S = 3 3S. Vậy bất dẳng thức (8.4) được chúng minh. Bổ đề 8.1 (Bất đẳng thức Hadwiger-Finsler 1, 1939). Trong mọi tam giác ABC có bất đẳng thức √ a2 + b2 + c2 ≥ ( a − b)2 + (b − c)2 + (c − a)2 + 4 3S. (8.5) Chứng minh. Đặt x = p − a, y = p − b, z = p − c. Dễ dàng thấy rằng p = x + y + z, a2 − (b − c)2 = 4( p − b)( p − c) = 4yz, b2 − ( a − c)2 = 4( p − a)( p − c) = 4xz, c2 − ( a − b)2 = 4( p − a)( p − b) = 4xy. Do đó bất đẳng thức (8.5) tương đương với bất đẳng thức q 4( xy + yz + zx ) ≥ 4 3( x + y + z) xyz. Bình phương hai vế bất đẳng thức trên ta được ( xy + yz + zx )2 ≥ 3( x + y + z) xyz, hay ( xy)2 + (yz)2 + (zx )2 ≥ ( xy)(yz) + (yz)(zx ) + (zx )( xy). Bất đẳng thức cuối cùng hiển nhiên đúng. Dấu đẳng thức xảy ra khi và chỉ khi x = y = z ⇔ p − a = p − b = p − c ⇔ a = b = c. Bổ đề được chứng minh. 17
  18. Hội thảo Khoa học, Sầm Sơn 28-28/09/2019 Bổ đề 8.2 (Bất đẳng thức Gordon 1966). Trong mọi tam giác ABC có bất đẳng thức √ ab + bc + ca ≥ 4 3S. (8.6) Chứng minh. Bất đẳng thức √ a2 + b2 + c2 ≥ 4 3S + ( a − b)2 + (b − c)2 + (c − a)2 tương đương với √ ab + bc + ca ≥ 4 3S + ( a2 + b2 + c2 − ab − bc − ca). √ Vì a2 + b2 + c2 − ab − bc − ca ≥ 0 nên ab + bc + ca ≥ 4 3S. Dấu đẳng thức xảy ra khi và chỉ khi a=b=c. 9 Bất đẳng thức mới liên quan đến đường phân giác và đường trung tuyến 9.1 Giới thiệu Cho tam giác ABC, ta ký hiệu A, B, C là các góc của tam giác, a, b, c là độ dài các cạnh tương ứng, ký hiệu wa , wb , wc và m a , mb , mc tướng ứng là các đường phân giác góc A, B, C và các đường trung tuyến. Gọi R, r và p tương ứng là bán kính đường tròn ngoại tiếp, đường tròn nội tiếp, và nửa chu vi của tam giác, ký hiệu S là diện tích của tam giác ABC. Các đường phân giác và đường trung tuyến của tam giác có nhiều tính chất thú vị. Nói riêng, bất đẳng thức về đường phân và đường trung tuyến là chủ đề rất hấp dẫn và đóng vai trò quan trọng trong nghiên cứu hình học. Một lượng lớn các kết quả liên quan có thể xem trong các chuyên khảo nổi tiếng [1, 5]. Trong [5] ( tr. 220) trình bày các bất đẳng thức sau p2 ≤ ∑ ma wa ≤ 3(2R2 + r2 ). (9.1) Đây là bất đẳng thức Janous mở rộng (theo [7]). Trong [5]( tr. 219) cũng trình bày bất đẳng thức ma 13 r ∑ wa ≥ 4 − 2R . (9.2) Mục tiêu chính của mục này là đưa ra một vài bất đẳng thức hình học liên quan đến đường phân giác và đường trung tuyến của tam giác, vận dụng những bất đẳng thức đó, có thể cải tiến các bất đẳng thức (9.1) và (9.2). 18
  19. Hội thảo Khoa học, Sầm Sơn 28-28/09/2019 9.2 Một số bổ đề Để chứng minh kết quả chính của chúng tôi, chúng tôi sẽ sử dụng các kết quả sau. Bổ đề 9.1. Trong tam giác ABC, ta có S2 m2a w2a = p2 ( p − a)2 + ( b − c )2 . (9.3) ( b + c )2 Chứng minh. Theo công thức đường ptrung tuyến và đường phân giác của tam giác ABC 1 2 bcp( p − a) : m2a = (2b2 + 2c2 − a2 ) và wa = , ta dễ dàng tính được 4 b+c 1 m2a = p( p − a) + (b − c)2 , (9.4) 4 và p( p − a) w2a = p( p − a) − ( b − c )2 . (9.5) ( b + c )2 Từ (9.4) và (9.5), ta có ( p( p − a))    2 2 1 2 2 m a wa = p( p − a) + (b − c) p( p − a) − (b − c) 4 ( b + c )2 1 p( p − a)(b − c)4 1 p2 ( p − a )2 = p2 ( p − a )2 − + p ( p − a )( b − c ) 2 − ( b − c )2 4 ( b + c )2 4 ( b + c )2 p( p − a)(b − c)2 = p2 ( p − a )2 [(b + c)2 − 4p( p − a) − (b − c)2 ] 4( b + c )2 p( p − a)(b − c)2 = p2 ( p − a )2 [(b + c)2 − 4(b + c + a)(b + c − a) − (b − c)2 ] 4( b + c )2 p( p − a)(b − c)2 2 = p2 ( p − a )2 [ a − ( b − c )2 ] 4( b + c )2 p( p − a)(b − c)2 = p2 ( p − a )2 4( p − b)( p − c) 4( b + c )2 p( p − a)( p − b)( p − c) = p2 ( p − a )2 ( b − c )2 ( b + c )2 S2 = p2 ( p − a )2 + ( b − c )2 . ( b + c )2 Bổ đề 9.2. Trong 4 ABC, ta có ( b + c )2 p2 + 10Rr + r2 ∑ bc = 2Rr . (9.6) Chứng minh. Ta có ( b + c )2 1 1 ∑ bc = abc ∑ a(b + c)2 = abc ∑ a(2p − a)2 1 = (4p2 ∑ a − 4p ∑ a2 + ∑ a3 ). abc 19
  20. Hội thảo Khoa học, Sầm Sơn 28-28/09/2019 Dựa theo các đẳng thức trong [5] (trang 52), ta có ∑ a2 = 2( p2 − 4Rr − r2 ), ∑ a3 = 2p( p2 − 6Rr − 3r2 ), ∑ a = 2p, abc = 4Rrp, nên ta thu được (9.6). 9.3 Kết quả chính Định lý 9.1. Trong tam giác ABC, ta có S2 p( p − a) ≤ m a wa ≤ p( p − a) + ( b − c )2 . (9.7) 2p( p − a)(b + c)2 Dấu bằng xảy ra khi và chỉ khi b = c. Chứng minh. Theo Bổ đề 9.1 ta dễ dàng tính được m2a w2a ≥ p2 ( p − a)2 , từ đó ta thu được vế trái của (9.7). Mặt khác, theo Bổ đề 9.1, ta có S2 m2a w2a = p2 ( p − a)2 + ( b − c )2 ( b + c )2 2 S2 ( b − c ) 2 S4 ( b − c ) 4  = p( p − a) + − , 2p( p − a)(b + c)2 4p2 ( p − a)2 (b + c)4 2 S2 ( b − c ) 2  ≤ p( p − a) + . 2p( p − a)(b + c)2 Suy ra điều phải chứng minh. Hệ quả 9.1. Trong tam giác ABC, ta có 1 m a w a ≤ p ( p − a ) + ( b − c )2 . (9.8) 8 Chứng minh. Theo Định lý 9.1, ta chỉ cần chứng minh S2 1 < (9.9) 2p( p − a)(b + c)2 8 hay tương đương với 4S2 − p( p − a)(b + c)2 < 0 ⇔ p( p − a)[4( p − b)( p − c) − (b + c)2 ] < 0 ⇔ p( p − a)[ a2 − (b − c)2 − (b + c)] < 0 ⇔ p( p − a)[( a + b + c)( a − b − c) − (b − c)2 ] < 0. Vì a < b + c nên (9.9) đúng. 20
nguon tai.lieu . vn